4
$\begingroup$

Let $G$ be a compact group and let $V$ be a finite-dimensional complex vector space with a Hermitian inner product. Let $$\alpha : G \to U(V)$$ be an irreducible unitary representation. Then this gives rise to an action $\beta$ of $G$ on $\mathrm{End}_{\mathbb{C}}(V)$ given by $$ \beta_x(T) = \alpha_x T \alpha_x^*. $$

If $P = P^* = P^2$ is a rank-one projection in $\mathrm{End}_{\mathbb{C}}$, then so is $\beta_x(P)$ for each $x \in G$, i.e. the whole orbit consists of projections. I would like to find conditions which guarantee that these projections span the entire algebra of endomorphisms of $V$.

Question:

Is it known when

$$ \mathrm{span} \{ \beta_x (P) \mid x \in G \} = \mathrm{End}_{\mathbb{C}}(V)? $$

Some partial data:

  1. If $G$ is a compact Lie group and $P$ is the projection onto the span of the highest (or lowest) weight vector, then the condition holds.
  2. If $G = SU(2)$ and $V$ is the three-dimensional irreducible representation, this can fail for some $P$. For instance, take $P$ to be the projection onto the (one-dimensional) space of vectors of weight $0$.

I am mainly interested in the situation for finite groups and for compact Lie groups, but if there is anything more general out there I would of course be interested in that as well.

$\endgroup$
6
  • $\begingroup$ If I understand your question, it's as to when $T$ is a "cyclic vector" for $End(V) = V^\ast \otimes V$, not that that helps me so much. $\endgroup$ Oct 18, 2011 at 4:16
  • $\begingroup$ @Allen, yes, that is what I am asking. $\endgroup$
    – MTS
    Oct 18, 2011 at 5:01
  • $\begingroup$ There is something I don't understand in your question. The projections always lie in the proper subspace of Hermitian endomorphisms ($P^*=P$), so they can never span $End(V)$. $\endgroup$ Oct 18, 2011 at 16:08
  • $\begingroup$ @Claudio, that is not true. For instance if $P$ is Hermitian, $iP$ is anti-Hermitian. Any operator can be decomposed into a linear combination of Hermitian operators: $T = \frac{1}{2}(T + T^*) + i \frac{1}{2i}(T-T^*)$. $\endgroup$
    – MTS
    Oct 18, 2011 at 16:26
  • $\begingroup$ i.e. the Hermitian endomorphisms do not form a subspace. $\endgroup$
    – MTS
    Oct 18, 2011 at 16:27

0

Your Answer

By clicking “Post Your Answer”, you agree to our terms of service and acknowledge that you have read and understand our privacy policy and code of conduct.

Browse other questions tagged or ask your own question.